Questions STRATEGY

Ace your homework & exams now with Quizwiz!

Which manufacturing environment is best suited for the introduction phase of a product's life cycle? A. Job shop B. Engineer-to-order C. Flow D. Assembly line

A. Job shop The job shop layout allows for the incorporation of new product into existing work centers.

Strategies that lack stability over time and employee acceptance are the result of an organization not having which of the following? A. Big opportunities and eight accelerators B. Mission and vision C. Differentiation strategies D. Dependability and flexibility

B. Mission and vision. A business's mission, vision, and core values create a foundation for the organization's strategy by unifying leaders and employees behind a shared vision of the organization's principles and future. Without this foundation, strategies can lack stability over time and broad acceptance by executives, managers, and employees.

What factors are considered in determining industry attractiveness? A. Profitability and supplier collaboration B. Supplier collaboration and relative strength of seller versus rivals C. Profitability and cumulative competitive pressure D. Supplier collaboration and cumulative competitive pressure

C. Profitability and cumulative competitive pressure Profitability and cumulative competitive pressure are factors considered when determining an industry's attractiveness.

What is a good policy concerning a company's internal rate of return (IRR)? A. Approve the project if the IRR exceeds the cost of capital. B. Exceed the costs of material, labor, and overhead prior to approval. C. Set margins based on competitive analysis. D. Reverse-engineer competing products whenever possible.

A. Approve the project if the IRR exceeds the cost of capital. The IRR is the rate of compound interest at which the company's outstanding investment is repaid by the proceeds from a project.

In which of the following production environments is it most unlikely that planning bills are used for master scheduling? . A. Engineer-to-order B. Make-to-stock C. Make-to-order D. Assemble-to-order

A. Engineer-to-order Engineer-to-order environments will have least consistency from one order to another, making it difficult to use planning bills.

What are three of the broad areas necessary to address how well the strategy execution process is performing? A. Financial, customer, employees B. Customer, backorder/backlog, critical ratio C. Operations, financial, schedule D. Quality, financial, employee turnover

A. Financial, customer, employees Accurate and timely information about daily operations is essential if managers are to gauge how well the strategy execution process is proceeding. Information systems need to cover five broad areas: Customer data Operations data Employee data Supplier/strategic partner data Financial performance data

Which of the following describes the Global Reporting Initiative (GRI) guidelines concerning a company's environmental performance? A. Framework around which a company can set its own goals B. Set of rules that the U.S. Environmental Protection Agency (EPA) enforces for U.S.-based companies and their tier 1 suppliers C. Mandatory set of European Environment Agency (EEA) rules that EU-based companies are required to follow D. Preliminary set of rules that the U.S. Environmental Protection Agency (EPA) is considering codifying into law

A. Framework around which a company can set its own goals The GRI framework sets out principles and performance indicators that organizations can use to measure and report their human rights, labor, environment, and anti-corruption practices and outcomes.

A job shop environment includes which of the following? A. Functional layout, variable routings, job costing B. Functional layout, variable routings, dedicated machines, job costing C. Variable routings, dedicated machines, job costing D. Functional layout, variable routings, dedicated machines

A. Functional layout, variable routings, job costing Dedicated machines are typically found in a flow ship, while multi-purpose machines are found in a job shop.

Many organizations use a balanced scorecard to create key performance indicators for their strategies. What feature of the balanced scorecard approach makes it attractive for this purpose? A. Helps define multiple perspectives on the organization's strategic goals B. Provides detailed performance objectives down to the operations unit level C. Demonstrates compliance with general accounting principles D. More valid from the perspectives of external stakeholders

A. Helps define multiple perspectives on the organization's strategic goals A balanced scorecard represents external stakeholders' needs, financial objectives, operational or process needs, and future development needs.

If an organization has implemented a sustainability initiative but it has not produced the desired results, what step should the organization take next? A. Implement an audit. B. Prepare a business case for the plan. C. Educate the organization about the goals of the plan. D. Revise the plan's objectives.

A. Implement an audit. The next step the organization takes should be to implement an audit. All of the other options should have preceded implementation of the sustainability initiative.

A discrete manufacturer's marketing strategy is to be the low-cost, high-quality, high-volume, rapid-delivery supplier. Which of the following manufacturing processes would best fulfill this marketing strategy? A. Line B. Batch C. Cellular D. Jobbing

A. Line A line process has high volume and high quality but usually narrow effective variety.

Which of the following types of manufacturing processes would most likely be characterized by a narrow range of standard products, low direct labor costs, and a high level of technical support for manufacturing? A. Line B. Batch C. Jobbing D. Project

A. Line Line is a type of manufacturing process used to produce a narrow range of standard items with identical or highly similar designs. Production volumes are high, production and material-handling equipment is specialized, and all products typically pass through the same sequence of operations.

When implementing new productivity technology, a significant risk is that the application: A. May not work as planned, be done on schedule, or stay within budget. B. May not meet paying customers' needs. C. May not be operable without internal or external experts. D. May not have the necessary documentation for training and implementation.

A. May not work as planned, be done on schedule, or stay within budget. The risk with implementing new productivity technology is that it may not work and the project may not be done on time or within budget.

Which investment analysis tool accounts for the time value of money? A. Net present value B. Residual income C. Return on investment D. Payback period

A. Net present value Of the choices, only net present value uses the discounted rate of the original investment. Return on investment does not reflect the time value of money. Payback period shows how long it will take for the initial investment to be recouped. Residual income uses a minimum required rate of return in its analysis of gain from capital investment.

Which of the following is the best performance measurement to use to obtain higher profitability? A. On-time completion to schedule B. Utilization C. Volume production D. Productivity

A. On-time completion to schedule On-time completion to schedule means that what is scheduled is available when it is required, at a quality acceptable to the customer.

What is the Global Reporting Initiative (GRI)? A. Organization that pioneered a sustainability reporting framework B. Guidelines for a global enterprise to use in reporting on production C. Supplier scheduling tool for communicating supply status D. Internet-based enterprise resources planning system

A. Organization that pioneered a sustainability reporting framework The GRI is a network-based organization that pioneered the world's most widely used sustainability reporting framework.

In a continuous flow manufacturing environment, such as a grain milling operation, what inventory type is most likely to require safety stock? A. Raw materials B. Semifinished products C. WIP D. Finished goods

A. Raw materials In a continuous flow process, amounts of WIP, product in a semi-finished state, and finished goods are known and would not require safety stock. Raw materials, however, might be subject to interrupted supply, which would seriously affect production.

To demonstrate support for a quality improvement process, management should provide: A. leadership. B. consultants. C. discipline. D. financial backing.

A. leadership. Leadership, encouragement, and recognition are included in the role of management.

A critical role that a shop floor supervisor can play in health, safety, and environment (HSE) compliance is ensuring ease of access to A. safety data sheets. B. regulatory requirements. C. senior management for complaints. D. GRI sustainability reports.

A. safety data sheets. Managing safe working conditions and practices is a primary HSE concern, and one of the most important responsibilities is to ensure ease of access to safety data sheets (SDS).

The strategic risk plan must have go/no-go guidelines for: A. switching from problem mitigation to recovery. B. customer contracts. C. the legal community. D. make-or-buy decisions.

A. switching from problem mitigation to recovery.. The strategic risk plan asks, "At what point do we reach the limit of avoidance and mitigation strategies before we start to rely on recovery strategies?"

In corporate social responsibility, three measures of company performance are the economic dimension, the social dimension, and: A. the environmental dimension. B. the political dimension. C. the cultural dimension. D. the diversity dimension.

A. the environmental dimension. The triple bottom line measures the economic, social, and environmental consequences of a firm's activities.

A company has redesigned its packaging and is saving a large amount of plastic annually. This helps the environment, lowers costs, and makes kit assembly easier for workers, so it is best understood as an example of: A. the sustainable "sweet spot." B. cost improvement efforts. C. common sense. D. a Global Reporting Initiative (GRI) requirement.

A. the sustainable "sweet spot." Saving a large amount of plastic annually is a sustainable strategy to benefit the environment, the organization's economic interests, and helps make workers jobs easier, so it improves each area of the triple bottom line, which is called the sustainability "sweet spot."

A new printing press costs $50,000 today and will bring in net cash inflows of $10,000 the first year, $15,000 the second year, $20,000 the third year, and $25,000 the fourth year. Assuming a 6% cost of capital, the present value (PV) of these cash inflows is $9,433.96 the first year, $13,349.95 the second year, $16,792.39 the third year, and $19,802.34 the fourth year. What is the net present value (NPV)? A. $7,356.34 B. $9,378.64 C. $20,000.00 D. $57,356.34

B. $9,378.64 Since the present values of the future cash inflows are already calculated, add these PVs together to find their sum and then subtract the initial investment of $50,000: $59,378.64 - $50,000.00 = $9,378.64

In a company with a 12% internal rate of return, a capital budget item costs $18,000 and has annual savings of $2,500 in labor and $500 in annual power use. What would be the simple payback period in years? A. 5 years B. 6 years C. 8.3 years D. No enough information is provided to calculate the payback period.

B. 6 years $18,000/$3,000 = 6 years. The internal rate of return is not needed to calculate this answer.

What is a benefit of developing strategic alliances with key suppliers? A. Better response time from suppliers B. Ability to form long-term contracts C. Elimination of extra suppliers D. Vertical integration

B. Ability to form long-term contracts Strategic alliances are the basis for long-term contracts, which provide multiple benefits for both parties.

In an emerging market for a new product, financially strong companies pose which of the following risks to initial frontrunners? A. Increasing the complexity of products B. Acquiring operations of weaker competitors C. Increasing the velocity of distribution channels D. Reducing the barriers to entry

B. Acquiring operations of weaker competitors Financially strong organizations have the ability to reduce competition by buying weaker marketplace rivals.

What are four viable mitigation options for a situation in which a failure becomes external? A. Replacing, reworking, refunding, having a dinner meeting with purchasing agent B. Apologizing, refunding, reworking, providing compensation C. Refunding, replacing, reworking, threatening a lawsuit D. Refunding, reworking, replacing, canceling all business

B. Apologizing, refunding, reworking, providing compensation Failure recovery strategies can involve a wide range of activities such as apologizing, refunding monies, reworking the product or service, or providing compensation.

Analysts must be aware of which of the following when assessing potential points of weakness in a risk assessment for a technology-based business? A. Outsourcing is a viable alternative to increase capacity and provide flexibility to customers. B. At any moment, a competitor may experience a breakthrough and take market share. C. A solidly composed patent will give the business many years of competitive protection. D. Colleges and universities are often available for cutting-edge research and development.

B. At any moment, a competitor may experience a breakthrough and take market share. A technology-based supplier of high-tech coatings was caught by surprise when its competitor introduced a new coating technology, enabling it to produce product at significantly better levels of cleanliness.

A CEO suggests some additions to the company's automated dashboard—a number of financial measures as well as information on customer complaints, manufacturing cycle time by product group, and year-to-date spend on education and training. What is this view commonly called? A. Product positioning perspective B. Balanced scorecard C. House of holistic perspectives D. Balance sheet

B. Balanced scorecard A balanced scorecard is a list of financial and operational measurements used to evaluate organizational performance. It may include customer, business process, financial, and innovation and learning perspectives. It formally connects overall objectives, strategies, and measurements.

What do companies often use to sustain financial objectives matched with strategic objectives? A. Ratio of earnings per share and stock price B. Balanced scorecard C. Total market capitalization value of the company D. Balance sheet

B. Balanced scorecard The most widely used framework for keeping financial objectives matched with strategic objectives is the balanced scorecard.

What formula is used to calculate the cash-to-cash cycle time? A. Accounts Receivable - Accounts Payable B. Inventory Days + Accounts Receivable Days - Accounts Payable Days C. Average Accounts Payable Days D. Inventory Days + Accounts Payable Days - Accounts Receivable Days

B. Inventory Days + Accounts Receivable Days - Accounts Payable Days

Practices or guidelines that minimize the impact of an operation on the environment are part of a company's policy on: A. strategic diversity. B. sustainability. C. the reverse supply chain. D. hazardous material (HAZMAT).

B. sustainability. Having a sustainability policy means that an organization's activities and operations will not compromise the needs of future generations.

Management is primarily a skill that keeps a complicated system of people and processes functioning well. Which of the following is true of leadership? A. It involves managing the managers. B. It is a set of processes that creates organizations or adapts them to significantly changing circumstances. C. It is involved in looking to the future using market trends, demographic change, and worldwide political events. D. It is more oriented toward working with marketing and corporate financiers.

B. It is a set of processes that creates organizations or adapts them to significantly changing circumstances. Leadership is a set of processes that creates organizations in the first place or adapts them to significantly changing circumstances.

In manufacturing, what is the cash conversion cycle? A. Length of time for a new product to generate profits B. Length of time from purchase of raw materials to collection of accounts receivable C. Ratio of accounts receivable for shipped end items to accounts payable for raw materials D. Average time open to close for accounts receivable

B. Length of time from purchase of raw materials to collection of accounts receivable The cash conversion cycle in manufacturing is defined as the length of time from the purchase of raw materials to the collection of accounts receivable.

An organization consistently wins awards for the designs of its products, but customer satisfaction scores don't seem to reflect this product superiority. What is likely the root cause of this issue? A. Order winners becoming order qualifiers B. Poor service provision C. Fickle customers D. Even better competitor products

B. Poor service provision The combination of the product and its associated services is sometimes called the product-service package to indicate that the two are inseparable, especially in the mind of the customer. The customer may like a product itself, but if it has poor associated services, the customer will still likely not be satisfied.

Which of the following generic performance objectives should be considered first when trying to identify what to measure? A. Bottom-line financial measures such as net profit or earnings before interest and taxes (EBIT) B. Quality, speed, dependability, flexibility, cost C. Only the major milestones as displayed in a project PERT or Gantt chart D. Cost, quality, schedule

B. Quality, speed, dependability, flexibility, cost In deciding what to measure in an organization, quality, speed, dependability, flexibility, and cost should be considered first.

Considering sustainability as a strategic initiative offers which of the following advantages? A. The organization can avoid having to make sustainability policies. B. Some larger initiatives can be planned in addition to addressing "low-hanging fruit." C. Investments in sustainable initiatives can be decentralized and unique. D. Senior management can delegate the process to an "army of volunteers."

B. Some larger initiatives can be planned in addition to addressing "low-hanging fruit." Taking a strategic approach to sustainability promotes a more proactive perspective such as attention to future problems and opportunities. This can help avoid a problem in corporate sustainability programs: restricting the organization's attention to "low-hanging fruit" that offers quick, but in the long term, small results.

An organization is working to earn the ASCM Enterprise Certification. What can make it more difficult to satisfy the certification's checklist related to plan, source, make, deliver, return, and enable? A. The organization's culture places high value on sustainability and so has high expectations for its supply chain. B. The organization has a highly complex supply chain. C. The organization is at the "civil" stage of corporate social responsibility maturity. D. The organization has a locally adaptive approach to globalization and supply chain management.

B. The organization has a highly complex supply chain. As the supply chain grows more complex, it becomes more difficult to know with certainty suppliers' and distributors' social and environmental conditions. Maintaining a sustainable supply chain may require use of third parties to assess and monitor supplier conditions.

If a customer or service provider is given less leeway to customize a service, what is one tradeoff? A. The degree of contact between the customer and the service provider must be higher. B. The sales opportunity is lower. C. Production efficiency is lower. D. The perceived value of the service must be higher.

B. The sales opportunity is lower Per the service design matrix, reducing opportunities for customization will reduce the likelihood of making a sale. However, production efficiency will be higher.

What is a key purpose of life cycle costing? A. To identify exactly where a product lies in contrast to its product launch B. To identify all environmental and social impacts generated from the production of a particular item C. To identify the total form, fit, and function over the entire history of the product D. To identify the point at which a sustained decline in demand develops

B. To identify all environmental and social impacts generated from the production of a particular item Life cycle costing attempts in part to identify all environmental and social impacts generated from the production of a particular item.

Which is an example of design for sustainability? A. Developing the next new solar panel so it can be released as the current model is starting to slow in sales. B. Working with supply chain members to ensure that agricultural practices eliminate toxic chemicals C. Deconstructing a competitor's new LED lighting fixture to determine how to build one just like it as a "me too" product D. Developing a subcomponent that can be attached using the same standard nuts and bolts used in the rest of the product

B. Working with supply chain members to ensure that agricultural practices eliminate toxic chemicals Nike is working to eliminate toxic chemicals from its supply chain, and this is an example of design for sustainability. While the other answers may have something to do with sustainability, designing for standard nuts and bolts would be better described as design for manufacturability. Deconstructing an LED lighting fixture is an example of reverse engineering. Developing a follow-on product for a product that is in decline is an example of product life cycle planning.

Current trends are essentially forcing companies that use outsourcing to ensure that all suppliers conform to: A. EDI (electronic data interchange) protocols. B. appropriate corporate social responsibility practices. C. USMCA regulations. D. Electronic Product Code (EPC) standards.

B. appropriate corporate social responsibility practices. When a company chooses to outsource any part of its operations, it must consider quality, speed, dependability, flexibility, and cost and must ensure that suppliers conform to appropriate corporate social responsibility (CSR) practices.

The current ratio is used to appraise the ability of a company to: A. sustain its current rate of sales. B. satisfy its current debt out of current assets. C. attain its current year-end earnings-per-share goal. D. issue more founders' stock.

B. satisfy its current debt out of current assets. The current ratio, used to appraise the ability of the company to satisfy its current debt out of current assets, is calculated as follows: (Current Assets/Current Liabilities)

An operation's actual run time is 300 minutes and its planned run time is 400 minutes. The operation's actual quantity produced is 900 units but its target quantity produced is 1,000 units. Of the 900 units produced, 810 were produced right the first time and 90 required rework. What is the overall equipment effectiveness (OEE)? A. 6.75% B. 54.68% C. 60.75% D. 76.67%

C. 60.75% The OEE is the availability times the performance times the quality. Availability is actual run time divided by planned run time, or 300 minutes/400 minutes = 0.75. Performance is actual quantity produced divided by target quantity produced, or 900 units/1,000 units = 0.9. Quality is the quantity produced right the first time divided by the total quantity produced, or 810 units/900 units = 0.9. OEE = 0.75 x 0.9 x 0.9 = 0.6075, or 60.75%.

A manufacturing company implementing a focused factory strategy has likely come to what conclusion? A. A focused factory will gain significantly from bill-of-material modularization. B. A focused factory is more likely to recover from a market downturn. C. A focused factory will outperform a plant attempting a broader set of objectives. D. A focused factory will likely suffer higher costs due to under-capacity utilization if more diversified.

C. A focused factory will outperform a plant attempting a broader set of objectives. The focused factory idea holds that a plant focusing on a narrow range of products aimed at a particular market will outperform a plant attempting a broader set of objectives.

Which of the following performance measures is of no use to the management of a make-to-stock company with regard to master scheduling and sales and operations planning? A. Order completion per the master schedule B. On-time shipment per the promise date C. Achievement of the backlog plan D. Adherence to the inventory plan

C. Achievement of the backlog plan A make-to-stock environment uses an inventory plan, not a backlog plan.

The automaker Toyota turned societal concerns for fuel efficiency and automobile emissions into an industry-leading hybrid car strategy that enabled them to gain early market leadership. What is key to Toyota's strategy in this initiative? A. Differentiating itself from major competitors at this time B. Response to market needs to shape product designs C. Aligning competencies with social benefit for competitive advantage D. Awareness of need to mitigate harm from value chain activities

C. Aligning competencies with social benefit for competitive advantage This is a proactive response to an opportunity, using capabilities to create benefits.

Which of the following interfaces deals primarily with translating monetary units into product families for planning purposes? A. Sales and operations planning to master scheduling B. Strategic planning to business planning C. Business planning to sales and operations planning D. Master scheduling to detailed scheduling and planning

C. Business planning to sales and operations planning The business plan is in monetary units, while the sales and operations plan uses aggregate product family planning.

Which of the following environments would have the most predictable throughput? A. Assembly line B. Intermittent C. Continuous process D. Job shop

C. Continuous process A continuous flow would have a predictable, steady throughput.

A large manufacturer/distributor is working to reduce water use in its production processes and to improve the water supply in emerging supply markets. What is this an example of? A. Migrating environmental standards B. Getting in front of an issue that is likely to affect corporate image C. Design for sustainability D. Regulations working for the greater good

C. Design for sustainability A manufacturer/distributor working to reduce the water intensity of its production processes and to improve water resources in developing countries is focused on design for sustainability.

Which of the following process choices is likely to simplify the sales and operations planning process? A. Functional B. Batch C. Line D. Lot-for-lot

C. Line Lines produce at a rate that can be converted into a monthly supply quantity. The other responses may all have significant planning variation by month.

Which term is defined as the actual output of production compared to the actual input of resources? A. Utilization B. Input/output control C. Productivity D. Efficiency

C. Productivity Productivity is an overall measure of the ability to produce a good or a service. It is the actual output of production compared to the actual input of resources.

After receiving criticism from environmentalists concerning the negative side effects of excessive packaging, a major retailer decides to improve the carbon footprint of its supply chain in part by mandating that suppliers use smaller packaging. This has the side effect of enabling the retailer to put more product on its shelves. Which is a likely result of this change? A. The higher numbers of products carried and sold will result in a worse overall carbon footprint. B. The suppliers will be unwilling to comply with this mandate because it will cost them money in the long run. C. Reducing the supply chain carbon footprint will result in positive environmental press. D. The increase in profits from more inventory on shelves will offset the negative press that will likely continue.

C. Reducing the supply chain carbon footprint will result in positive environmental press. Sometimes unintended consequences have positive outcomes. For example, Walmart is recognized for its efforts to reduce packaging and improve the carbon footprint of its supply chain following negative press on the subject. However, they are pursuing footprint reductions largely because doing so allows them to put more product on the shelf, which means more products to sell.

There are two generally recognized types of errors that can occur when responding to risk. In the first, the organization takes action, but it is an incorrect action. What is the second? A. The organization postpones a decision while operations deteriorate. B. The organization halts all operations to gather more data. C. The organization does not take action when it should have. D. The organization continues to support an incorrect action when evidence suggests that it should not.

C. The organization does not take action when it should have. Two basic types of errors an organization can make when responding to risk are: Taking an action that is incorrect. Not taking action when they should have.

Which of the following performance measurements is likely to be the most important to operations managers of a product line with medium volume and a short life cycle? A. Capacity utilization B. Standard error of the forecast C. Time to market D. Number of schedule changes

C. Time to market Short life cycle production is usually focused on products that need to get to market quickly. Production is medium volume, so the smaller lot sizes enable managers to quickly schedule and reschedule production.

On a matrix that has service production volume on one axis and service variety on the other, which is the best example of a low-variety, high-volume service? A. Standard dental cleaning visit B. Online photo printing service C. Vehicle oil change service D. Phone calls to sales prospects

C. Vehicle oil change service The oil change service has the lowest variety. Photo printing, for example, typically has many options. Oil change services are also high-volume services compared to other automotive maintenance options.

In order to inspire and motivate people, successful leaders will: A. have charisma that naturally inspires people to follow. B. have their biography written. C. create the conditions that make goals achievable. D. use their political connections to further their goals.

C. create the conditions that make goals achievable. Motivation is one thing, but competent strategic leadership is the necessary foundation for success. The job of the leader—the strategist—is to also create the conditions that will make the push possible, to ensure that the team has the resources they need.

All of the following are considerations in the adoption of a process focus except: A. the type of technology used in the manufacturing process. B. the complexity of the manufacturing process. C. the book value of current production assets. D. the capital intensity of the manufacturing process.

C. the book value of current production assets. The book value of current production assets will have no effect on how the assets will be used in production.

In addition to a financial evaluation, assessment of capital investments should include taking which of the following actions? A. Reviewing available technologies B. Analyzing customer needs C. Reviewing organizational capabilities D. All of the above

D. All of the above Financial planners need to review all of these when assessing capital investments to ensure that the assets needed by the organization are available.

Which organization is pursuing a multidomestic strategy? A. A manufacturer enters into distribution agreements with dealer networks in multiple foreign market to sell goods produced in the manufacturer's country. B. A services provider acquires similar businesses around the world, changes their names to reflect the provider's brand, and standardizes policies and services sold. C. A cereal manufacturer buys similar businesses in other countries. The manufacturer invests in improving the acquisitions' costs. Products remain the same. D. A manufacturer makes multiple foreign acquisitions and produces components in different countries with standard processes, but products are assembled locally to appeal to local market preferences.

D. A manufacturer makes multiple foreign acquisitions and produces components in different countries with standard processes, but products are assembled locally to appeal to local market preferences. A multidomestic strategy allows for integration of capabilities and customization to local practices and tastes.

An effective quality process for product design includes which of the following activities? A. Ensuring that the design is complex enough that it cannot be duplicated by competitors B. Determining the split of appraisal costs vs. prevention costs C. Aligning design features with demographic data for sales information D. Analyzing the product's or service's sensitivity to the environment

D. Analyzing the product's or service's sensitivity to the environment Analyzing the materials and processes used and their sensitivity to the environment will help the company achieve sustainability goals. Simple, standardized process designs are best.

What ratio can be used to determine a company's ability to pay its liabilities by converting assets in the near term? A. Accounts receivable turnover B. Debt ratio C. Inventory turnover D. Current ratio

D. Current ratio The current ratio reflects the ability to pay debts from assets that can be quickly converted to cash, unlike long-term assets or inventory, which is reflected in the debt ratio. Accounts receivable indicates the organization's ability to collect payments from customers, and inventory turnover indicates the speed with which assets are converted into revenue.

What are the three components of sustainability? A. Economic, environmental, ecological B. Social, environmental, regulatory C. Social, economic, regulatory D. Economic, environmental, social

D. Economic, environmental, social Sustainability includes elements of protecting the environment, sustaining the future economy, and being a responsible citizen.

Continuous production processes would be most appropriate for which of the following? A. Restoring furniture B. Mining of semiprecious gem stones C. Assembly of low-cost consumer electronics D. Generation of electric power

D. Generation of electric power Continuous production is a type of flow manufacturing, used for the production of high-volume standard products made in a continuous flow (such as gasoline) rather than as discrete units (such as cars and appliances).

What is a primary shortcoming of using the simple payback period method to justify capital budgeting expenditures? A. It does recognize the time value of money and is therefore more exact and realistic than using the accounting rate of return (ARR). B. It will likely not remain stable over the time period used. C. It is too simple to compute and therefore is subject to adulteration. D. It does not recognize the time value of money.

D. It does not recognize the time value of money. One of the primary shortcomings of using the payback period to justify capital budgeting expenditures is that it does not recognize the time value of money. The discounted payback period method does not have this disadvantage.

Companies that produce a narrow range of high-volume products will most likely choose which of the following types of processes? A. Batch B. Jobbing C. Project D. Line

D. Line Line manufacturing is defined as repetitive manufacturing performed by specialized equipment in a fixed sequence. It is suited to situations that involve a narrow range of high-volume products.

Focused factory operations include which of the following likely results? A. More expensive quality B. Higher average inventory C. More flexible operations D. Reduced material handling

D. Reduced material handling Improved quality and reduced inventory and material handling are valid benefits of a focused factory.

Managers must consider which of the following basic types of risks? A. Risks that cannot be avoided and risks that can be avoided B. Risk that have been fully defined and those that have been historically documented C. Risks with losses that difficult to quantify and risks that are variable due to a number of unknowns D. Risks associated with only negative results and risks that may have either negative or positive results

D. Risks associated with only negative results and risks that may have either negative or positive results Speculative risk differs from pure risk in that pure risk is associated with only negative results while speculative risk sees opportunity for both negative and positive outcomes.

What is a benefit of an effective work cell layout? A. Increased utilization B. Reduced setup time C. Reduced process variability D. Simplified materials handling

D. Simplified materials handling Benefits of the use of work cells (also called cellular layout) include that it minimizes material-handling distances and factory floor space requirements. Other benefits include reduced queue times, simplified control needs, and immediate feedback.

What is a key rule for operations performance measures? A. They must be consistent year after year. B. They must focus on a department's positive performance. C. They must be easily viewed on charts. D. They must align with the company's overall goals.

D. They must align with the company's overall goals. Measurements must align with the company's goals; they must change as the needs of the organization change.

A design engineering group has dismantled a new product from another supplier. This is an example of: A. unethical behavior. B. disintermediation. C. thievery. D. competitive analysis.

D. competitive analysis. Performing a competitive analysis of a competitor or its products allows a company to understand the competitor's strategies, capabilities, prices, and costs.

The five forces model: A. is used to determine the economic features of a market. B. determines what market positions rivals occupy. C. identifies market trends. D. considers the impact of buyers on competition.

D. considers the impact of buyers on competition. The five forces model consists of the following elements: rivalry among competitive sellers, buyer bargaining power, substitute products, supplier power, and new entrants into the marketplace.

The resource plan is typically stated in all of the following terms except: A. labor hours. B. machine hours. C. pounds. D. item numbers.

D. item numbers. Resource plans deal with the planning and managing of plant capacities. Pounds and hours are important capacity measurements. Item numbers are not relevant to capacity units of measure.

The quick asset ratio is a financial calculation best used to: A. assist with cash flow rate planning. B. measure a firm's equity valuation. C. learn if a company is top-heavy with regard to capital equipment. D. measure a firm's financial stability.

D. measure a firm's financial stability. The quick asset ratio, a measure of a firm's financial stability, is calculated as follows: (Current Assets - Inventory)/Current Liabilities

Scipa Automotive is a reputable automotive parts supplier with declining profit margins in its home market because of declining automobile sales and high labor costs. Scipa is considering acquiring a midsize parts manufacturer in another country, where a large automotive parts market is emerging. Scipa would transfer all manufacturing to this country. The business being considered for acquisition has weak management and technical competencies compared with Scipa's home-country levels. Which factor will likely be most influential in Scipa's decision? A. Access to a growing market B. Cutting costs of operations in its home country C. Protecting proprietary information D. Creating economies of scale

A. Access to a growing market Scipa wants to move production entirely, so economies of scale and cost cutting at home are not issues. Proprietary information can be an issue in expanding into another country, but that probably is not an issue in this case. Scipa is looking for more customers and could use its expertise to improve its investment in purchasing the company.

A company operates in a market characterized by low volume per product and a wide product range. It strives for a competitive edge by providing high-technology products designed specifically to meet the unique requirements of each of its customers. Which of the following characteristics most likely is the company's order-winning criterion? A. Design capability B. Price and quality C. Delivery speed D. Delivery reliability

A. Design capability The company's order-winning criteria are focused around the product manufacturing characteristics of low volume and wide product range. This means that customers buy the company's products because they are unique and are hard to find. They are also not sold in a mass market. As such, of the four criteria (design capability, delivery speed, delivery reliability, and price and quality) customers consider product design the most valuable order-winning criterion.

An organization develops a dual operating system consisting of a traditional organizational hierarchy and a network organizational structure. The latter develops "big opportunities." What is a typical role for the traditional organizational hierarchy? A. Executing strategic changes that are extensions of the current strategy B. Focusing more on leadership than management C. Focusing more on creativity than logic D. Executing strategic changes that require speed and agility

A. Executing strategic changes that are extensions of the current strategy The traditional hierarchical structure is good at running the day-to-day affairs of the organization, which includes executing the current strategy and continuously improving it.

An organization has determined that brand recognition is its most important strategic advantage for breaking into a foreign market. Which entry option would create the greatest risk to this advantage? A. Franchise B. Joint venture C. Subsidiary D. Export

A. Franchise When using franchising as an entry option to gain access to foreign markets, the franchisor gives up some control over its brand. Franchisees can weaken the brand in the new market by delivering poor quality or service. They may also alter the brand when it conflicts with local culture and customer preferences. This has the effect of diluting the power of the brand.

At what do the organizational structures commonly found in large and successful organizations tend to be effective and efficient? A. Functioning and executing B. Innovating and changing C. Such organizational structures tend to be neither effective nor efficient. D. Such organizational structures tend to be effective in most areas but not efficient.

A. Functioning and executing Traditional organizational structures that most large and successful organizations have are usually good at keeping the organization functioning and executing effectively and efficiently. However, these same structures tend to have lost the entrepreneurship and agile innovation ability that makes start-ups so great.

A new product line consisting of many individual products is projected to experience low volumes for the foreseeable future. What would be the appropriate manufacturing strategy for this situation? A. High involvement with people, low involvement with machines B. High involvement with machines, low involvement with people C. Highly scalable workforce D. Significant use of outsourcing for peak capacity seasons

A. High involvement with people, low involvement with machines Small-scale technologies combined with skilled staff or higher involvement of staff tends to be more flexible than investing in machines because higher involvement with machines requires more capital investment up front. The capital investment in dedicated machinery could be made later if demand increases.

Which of the following is a disadvantage of being an innovator in product design and technology? A. Higher research and development expenses may not increase market share or margins. B. A lengthy learning curve will be required of customers for correct product operation. C. Competitors may make poor-quality copies and destroy the market by making customers believe that all such products have poor quality. D. Early adopters will not aware of the product's unique features.

A. Higher research and development expenses may not increase market share or margins. Being an innovator in product design and technology is more costly than using previous designs or existing technology, so the increased research and development expense needs to be offset by higher market share or higher margins per sale or the organization will not be as profitable as competitors that spend less on innovation.

An organization has limited shop floor space to add new equipment and doesn't want to move or expand space this year. What would be the best way for it to still add agility and flexibility this year? A. Invest in new methodologies and replace some equipment with more versatile versions. B. Add new equipment in tighter configurations and train users on improving dexterity. C. Change from level to chase production planning and use hirings and layoffs more frequently. D. Change from chase to level production planning and add a shift.

A. Invest in new methodologies and replace some equipment with more versatile versions. Agility and process flexibility can be developed by making investments in methodologies such as lean as well as investing in equipment that enables flexibility.

What happens once the final draft of a company's strategy has been formulated? A. It is typically submitted to the company's board of directors for review and approval. B. It must be filed with the Securities and Exchange Commission. C. It must next be published in the company's annual report if the company is publicly traded. D. All stakeholders have approximately 30 days to fine-tune it.

A. It is typically submitted to the company's board of directors for review and approval. The strategy-making efforts of top managers are complemented by advice and counsel from the company's board of directors. Normally all major strategic decisions are submitted to the board for review, discussion, and official approval.

Which of the following production environments is most likely to have customer, forecast, and distribution demand? A. Make-to-stock B. Engineer-to-order C. Assemble-to-order D. Make-to-order

A. Make-to-stock Make-to-stock has forecast and distribution demand and possibly customer demand. The others typically do not have distribution demand.

Which of the following is typically an advantage of implementing a multidomestic/multicountry strategy rather than a global strategy? A. Marketing can be adapted to the local culture. B. Brand recognition is increased. C. Plants can be located where total costs are lowest. D. Economies of scale are achieved.

A. Marketing can be adapted to the local culture. In a multidomestic/multicountry strategy, a company varies its product offering and competitive approach from country to country to be responsive to differing markets. In a global strategy, product variations are minimized from country to country.

Which of the following criteria would be essential for the use of rough-cut capacity planning in an engineer-to-order company? A. Method of associating planned production with the amount of capacity required B. Successful implementation of a material requirements planning (MRP) system C. Accurate work-in-process inventory and finished goods D. Education of customers on capacity management concepts

A. Method of associating planned production with the amount of capacity required Managers and planners (not customers) have to be knowledgeable about capacity management and the use of computerized systems to link planned production and capacity availability. An MRP system is not required to perform this level of capacity management. In an engineer-to-order company, production is usually for unique, one-of-a kind products. This means that work-in-process inventories will be small.

A product has moved from regular production to spare parts supply. Which of the following factors is likely to become least important? A. Price B. Quality C. Customer service D. Delivery reliability

A. Price Once a product has moved from regular production to intermittent spare parts supply, customer service, delivery reliability, and quality must remain high. Service parts are usually critical to the customer and must be available at all times with perfect quality. Since the product is no longer considered "product," its price can become elastic and based on what the market will bear. Price, therefore, declines as an important factor, as customers will pay a premium when equipment goes down.

A company operates a strict pull system with its suppliers, with shipments every two, four, and eight hours. A third-party logistics supplier manages all incoming and outgoing shipments. Which of the following delivery performance measures is most likely to incorporate a tolerance to allow for uncontrollable conditions? A. Required time window B. Required advance shipping notice C. Required date D. Required carrier

A. Required time window Weather and traffic conditions cannot be controlled, so acceptable time windows need to be provided, including tolerances.

Which of the following guidelines should be used when selecting activities for performance measurement? A. The activity has a significant effect on the performance of the overall system, and the activity is measurable (objective and quantifiable). B. The activity has a significant effect on the performance of the overall system, the activity has a high potential for improvement, and the activity is measurable (objective and quantifiable). C. The activity has a high potential for improvement, the activity falls within the responsibility of the organization, and the activity is measurable (objective and quantifiable).= D. The activity has a significant effect on the performance of the overall system, the activity has a high potential for improvement, the activity falls within the responsibility of the organization, and the activity is measurable (objective and quantifiable).

A. The activity has a significant effect on the performance of the overall system, and the activity is measurable (objective and quantifiable). Performance measurement should focus on important activities, and only measurable activities can be measured.

Which of the following characteristics are desirable in a system measuring overall plant productivity? A. Visibility of multiple manufacturing resources, and identification of factors for improving throughput B. Visibility of multiple manufacturing resources, and presentation of data in discounted dollars C. Identification of factors for improving throughput, and detailed analysis of labor variances D. Visibility of multiple manufacturing resources, identification of factors for improving throughput, and detailed analysis of labor variances

A. Visibility of multiple manufacturing resources, and identification of factors for improving throughput Visibility of multiple manufacturing resources and identification of factors for improving throughput provide for the measurement of overall plant productivity.

What is vital to periodically evaluate regarding a breakthrough strategy designed to significantly increase inventory turnover? A. Whether the change has introduced unintended consequences such as to quality B. Whether inventory turnover is becoming too high and needs to be lengthened C. Whether enough pressure is being placed on functional areas that could further increase inventory turnover D. Whether specific goals should be set if progress on the strategy is starting to slow

A. Whether the change has introduced unintended consequences such as to quality Strategies and tactics need to be monitored to ensure that unexpected negative results have not been introduced. Increasing inventory turnover is a worthy goal, but it could promote taking shortcuts that harm quality and result in customer dissatisfaction.

Corporate goals identified using a top-down and bottom-up approach should be prioritized by using: A. a strengths, weaknesses, opportunities, and threats analysis involved with each goal. B. a Delphi forecast process where a panel of experts are engaged for this specific purpose. C. a Pareto analysis revealing that 80% of the potential gains will come from 20% of the opportunities. D. an activity-based costing process applied to each goal.

A. a strengths, weaknesses, opportunities, and threats analysis involved with each goal. By developing a strengths, weaknesses, opportunities, and threats (SWOT) analysis involved with each goal, the most important goals should move to the top.

In a supply network, the company is in the center, and it is surrounded by: . A. competitors, suppliers, and perhaps several tiers of customers. B. suppliers and their suppliers but never competitors. C. suppliers and perhaps several tiers of customers but never competitors. D. third- and fourth-party providers.

A. competitors, suppliers, and perhaps several tiers of customers. In a supply network, the company is surrounded by competitors, suppliers (and perhaps their suppliers), and perhaps several tiers of customers (e.g., distributors/wholesalers, retailers, and end customers).

When two business units are merged in order to exploit the opportunity to exchange R&D resources or use common manufacturing or distribution facilities, the companies are said to be sharing: A. economies of scope. B. intellectual property. C. design for manufacturability concepts. D. economies of scale.

A. economies of scope. Economies of scale are cost savings that accrue directly from a larger operation. For example, manufacturing costs may be lower in a larger plant than in a small plant. Economies of scope stem directly from strategic fit along the value chains of the related businesses.

A strategy to attain technology superiority in product development may lead to: A. market domination during the early stages of a product's life cycle. B. overly complex products containing unnecessary features. C. unnecessary complexity in the manufacturing process. D. copycat manufacturers who have no regard for intellectual property rights.

A. market domination during the early stages of a product's life cycle A principal offensive competitive strategy includes leapfrogging competitors by being the first to market with next-generation products. For example, with its Xbox, Microsoft led the market by 12 months, until Sony's PlayStation was released.

By approving projects that improve performance with its order qualifiers, a company is: A. not going to achieve high levels of competitive benefits. B. likely using the 80-20 rule for project prioritization. C. likely selecting improvement projects based on the theory of constraints. D. spending its money wisely.

A. not going to achieve high levels of competitive benefits. No matter how well the company performs at its qualifiers, it is not going to achieve high levels of competitive benefits.

In hopes of achieving strategic goals, executives may: A. provide resources for only those projects that facilitate accomplishing strategic goals. B. pit one department or group against another in hopes of instilling competition. C. engage specialists to lobby the government in hopes of attaining special tax breaks. D. initiate a public relations organization to advertise the company's strategic objectives.

A. provide resources for only those projects that facilitate accomplishing strategic goals. Compensation packages at many companies have increasingly rewarded executives for short-term performance improvements—most noticeably achieving quarterly and annual earnings targets and boosting stock prices by specified percentages. This can motivate the executives to take unwise business risks to boost short-term earnings. The focus on short-term performance has proved damaging to long-term company performance and shareholder interests.

A food producer and distributor of processed meats is considering purchasing a poultry business. This is an example of: A. vertical integration. B. green manufacturing. C. horizontal integration. D. sustainable business practices.

A. vertical integration. Vertical integration is the degree to which a firm has decided to directly produce multiple value-adding stages from raw material to the sale of the product to the ultimate consumer.

For some companies, value chain analysis has led to support processes such as design for sustainability, which is exemplified by: A. working to eliminate toxic chemicals from the supply chain. B. annual monitoring of the supply chain to ensure viability of parts and raw materials supply. C. supplier rating systems where all key suppliers are graded using common scorecards. D. ensuring that multiple suppliers are qualified for sourcing key components.

A. working to eliminate toxic chemicals from the supply chain. In its design for sustainability activities, Nestle, for example, is working to eliminate toxic chemicals from its supply chain. Pepsi and Coca-Cola are working to improve water resources in developing countries.

Production has determined that a particular order will be late due to a purchased part being past due. Which of the following scenarios is most likely to occur in a job shop environment? A. Material planning delays the order due date by one week. B. Another order takes its place in the dispatch sequence. C. The order is cancelled and re-created when the purchased part arrives. D. The order is completed up to the point where the part is needed.

B. Another order takes its place in the dispatch sequence. In a job shop, multiple jobs pass through functional departments in lots, and each lot may have a different routing. Multiple jobs would be using the same machinery, so if one could not be produced, another job would move ahead of it in sequence.

A diversified company is typically a collection of businesses. Which of the following is a strategy-making challenge for such a company? A. Gathering local marketing data for each industry and for the businesses within that industry B. Assessing multiple industry strategies, one for each industry arena in which the company operates C. Making small incremental changes in individual business units based on current market direction D. Becoming a decentralized organization that allows each business to formulate its own corporate strategy

B. Assessing multiple industry strategies, one for each industry arena in which the company operates Because a diversified company is a collection of individual businesses, the strategy-making challenge involves assessing multiple industry strategies, one for each industry arena in which the company operates.

What must operations do when the overall strategy is to compete with low-cost, no-frills products? A. Use outsourcing as much as possible regardless of supply chain complexity. B. Compromise from time to time with low-cost/low-quality component suppliers. C. Use technology. Robots that replace people don't look for breaks, vacations, or raises. D. Leverage low labor costs in countries with low labor rates without the assistance of sophisticated technology.

B. Compromise from time to time with low-cost/low-quality component suppliers. When a European car manufacturer acquired an existing automobile facility in an emerging nation just moving from central planning to market-driven planning, its hope was to build low-cost, no-frills cars. Leveraging existing platforms and a low cost design imperative, they succeeded. The facility was bare-bones with no robot technology, leveraging low labor rates.

What is an advantage of the lead capacity strategy? A. Low unit cost of production B. Maximizing of revenue potential C. Risk of permanent overcapacity D. High plant utilization

B. Maximizing of revenue potential As capacity will lead demand, utilization will be lower and production costs per unit will be higher than in a lag strategy. The risk of permanent overcapacity is high, so it is not an advantage.

Business-to-business customers have said that new products being rolled out need to have reasonable delivery reliability and price but insist on a product design that meets exacting precision requirements so it integrates with their product without costly rework. They do not care about delivery speed because they are able to place their orders far in advance. Which of the following strategies would be most likely to delight these customers? A. Consider product design to be the only order winner to focus on and accept that this will require a higher-than-average price, use slow and inexpensive shipping to compensate, and work on great delivery reliability using a make-to-stock strategy. B. Consider product design to be the only order winner to focus on, work to be above average at delivery reliability and price, and make-to-order and ship via slow and inexpensive methods when given sufficient lead time. C. Consider product design, delivery reliability, and price to be order winners to focus on so as to exceed expectations. D. Create a good design with best-in-class delivery reliability and a reasonable price by focusing on high production rates and inspections to ensure that only products that match the exacting specifications go out the door to the customer.

B. Consider product design to be the only order winner to focus on, work to be above average at delivery reliability and price, and make-to-order and ship via slow and inexpensive methods when given sufficient lead time. The combination of product and related services will delight customers when their order qualifiers are all met and their order winner qualities are kept as the highest priority. In this case, the organization does not try to make all things be order winners but focuses on great product design. They use slow shipping methods and make-to-order to keep their costs down and offer a price that keeps it as an order qualifier. Any extra costs required to ensure above-average delivery reliability could then be absorbed.

Which of the following describes core competencies? A. Not directly related to the company's products or markets B. Directly related to the company's products or markets C. Ideal candidates for cost improvement initiatives including outsourcing D. Quantifiable and dollarized in financial reporting

B. Directly related to the company's products or markets According to theAPICS Dictionary, core competencies are "bundles of skills or knowledge sets that enable a firm to provide the greatest level of value to its customers in a way that is difficult for competitors to emulate and that provides for future growth."

A supplier delivers books to electronic readers using 3G technology, eliminating distributors. What is this an example of? A. Advances in Wi-Fi B. Disintermediation C. Large area networking (LAN) D. Extensible markup language (XML)

B. Disintermediation Disintermediation is the process of eliminating an intermediate stage in a supply chain.

Greens is a fictional high-end gourmet shop that stocks and prepares unique foods for sale. Their clientele are a specific group of people who like to eat well, cook gourmet meals, or entertain guests. Greens offers a wide selection of gourmet products and focuses on excellence in customer service. They learn their customers by name, deliver products to their homes or businesses, and notify them when their favorites are available. Because of their business strategy, they have gained a loyal following with high customer satisfaction. How would you recommend Greens manage its competitors? A. Don't focus on current competitors. Think about future growth. B. Focus on maintaining a quality and service gap between Greens and its rivals. C. Increase spending on mass advertising to raise awareness throughout the market. D. Imitate the behavior of financially successful large chains.

B. Focus on maintaining a quality and service gap between Greens and its rivals. The best approach for Greens to take in managing its competitors is to focus on maintaining a quality and service gap. That is, Greens needs to ensure that its food offerings are of higher quality than its competitors.

One of an organization's core values is that "we share bad news early." Which is necessary to see this value in practice? A. The value is included in the mission and vision. B. Leaders and managers model the behavior. C. The mandate comes from the top. D. The value is published in the employee handbook.

B. Leaders and managers model the behavior. Core values serve as business and ethical principles for all members in the business, but they function only when they are mirrored in the behavior of leaders and managers.

What is an advantage of small-scale operations and facilities? A. Higher average production cost per unit B. Less overcapacity during scale-up C. Good fit for high-volume, low-variety operations D. Good fit for high-volume, low-variety operations

B. Less overcapacity during scale-up The smaller facility better matches demand during the scale-up period; a larger facility would have excess capacity until demand catches up with it. Higher unit cost is not an advantage. The other choices are advantages of large-scale operations and facilities.

Which of the following nonfinancial factors are critical to consider when setting up an offshore production facility? A. Local laws, quality of labor force, and differences in time zone. B. Local laws, quality of labor force, and local infrastructure. C. Local laws, differences in time zone, and local infrastructure. D. Quality of labor force, differences in time zone, and local infrastructure.

B. Local laws, quality of labor force, and local infrastructure. Understanding local laws, the quality of the local workforce, and local infrastructure are all critical considerations when searching for a location.

Which of the following is true of strategy development in most corporations? A. The CEO is the primary strategy architect, with little influence from others. B. Other senior executives help fashion major elements of the strategy. C. The business strategy is formulated first by department heads and then fine-tuned by the CEO and staff. D. Much "borrowing" through competitive benchmarking occurs.

B. Other senior executives help fashion major elements of the strategy. In most corporations, strategy is the product of more than just the CEO's handiwork. Typically, other senior executives—business unit heads, the CFO, and the vice presidents of production, marketing, HR, and other functional departments—have influential strategy-making roles and help fashion the chief strategy components.

What conditions are most favorable to a best-cost provider strategy? A. General increase in quality levels in category due to new technology B. Period of economic recession and declining household income C. Intense price-cutting tactics among category leaders D. Periods of economic energy and expansion of wealth

B. Period of economic recession and declining household income Periods of limited wealth create greater sensitivity to cost while customers still want some level of quality.

A company produces a high-volume electronic assembly using a series of dedicated production lines. This is an example of which process environment? A. Continuous B. Repetitive C. Job shop D. Batch

B. Repetitive The use of dedicated production lines for high-volume demand occurs in repetitive manufacturing.

What is the most accurate description of the make-or-buy decision? A. Current capacity should be excluded from the analysis. B. Short-term needs should be the primary consideration. C. The decision should not consider workforce stability. D. This type of decision should be revisited often.

B. Short-term needs should be the primary consideration. Make-or-buy decisions need to be re-evaluated as conditions that led to a particular decision change. Long-term costs, workforce stability, and capacity are all considerations, but not the primary.

Which of the following performance objectives refers to the time that has elapsed between the beginning and the end of the sales transaction? A. Flexibility B. Speed C. Dependability D. Quality

B. Speed There are five generic performance objectives: quality, speed, dependability, flexibility, and cost. Speed refers to the time that has elapsed between the beginning and the end of the sales transaction, and is focused on delivery lead time. The importance of speed varies depending on the industry - some industries will depend on getting products to customers as quickly as possible. Achieving speed requires the analysis of all transaction steps, in order to identify any opportunities to shorten the steps and/or the gaps between steps.

The IT function develops a strategy that calls for increasing use of cloud services. When this comes to the attention of the CEO, the CEO is unhappy. What is the most likely source of the problem? A. There is no robust test plan. B. The functional strategy is not aligned with the business strategy. C. No proven project management tactics are used during implementation. D. Top management is not directly involved and is not monitoring progress.

B. The functional strategy is not aligned with the business strategy. Some report that the typical IT project failure rate is between 35% and 75%. Often reasons cited are connected to managerial, implementation, or organizational factors. One of the main strategic issues is the degree of alignment and integration between IT strategy and the general strategy of the firm.

Success in operations drives better products and services, an increased range of product capability, better supply relationships, and heightened customer satisfaction due to high quality performance. Which of the following is true of all of these attributes? A. Once mastered, they should be relied upon to continue their high performance. B. They become strategic when taken from a holistic viewpoint. C. They are independent of strategy development, as they are all tactical. D. They may be relied upon to manage almost any new product design that could increase market share.

B. They become strategic when taken from a holistic viewpoint. Operational knowledge is vital to establishing an operations-based strategic advantage. A superior understanding of what one's processes can do at the limits of their capability allows the organization to better provide products and services, due to confidence in knowing for sure that the processes can deliver. Well-controlled processes will also have fewer errors and waste and therefore higher efficiency (lower costs, higher quality).

From the customer's perspective, what does delivery lead time consist of? A. Time from start of the order preparation to use of the product B. Time from receipt of the customer order to the delivery of the product C. Time from placing an order to use of the product D. Time from requirement for a product to receipt of the product

B. Time from receipt of the customer order to the delivery of the product Delivery lead time is the time from the receipt of a customer order to the delivery of the product.

What is the primary use of environmental scanning? A. To identify effective supply chains B. To identify threats and opportunities C. To understand current markets D. To understand competitors' strategies

B. To identify threats and opportunities Environmental scanning looks at external factors that represent either threats to the organization or opportunities for it. Identifying effective supply chains and understanding competitors and markets are not objectives of environmental scanning.

The possibility of maintaining or acquiring other diverse businesses will be facilitated by finding: A. an entity with a positive cash flow. B. a manufacturing strategic fit. C. a company with a similar strategic vision. D. an entity with solid assets.

B. a manufacturing strategic fit. When snowmobile maker Bombardier diversified into motorcycles, it was able to set up motorcycle assembly lines within the same manufacturing facility where it was assembling snowmobiles.

When strategically aligning operational capabilities with market needs, caution must be exercised so that: A. employee staffing is soft enough so that a good amount of overtime will be necessary. B. capabilities are not so tightly aligned as to offer little to no room for response flexibility. C. all capital projects are completely approved and aligned with the most optimistic market forecast. D. operations capabilities are 100% aligned with all market forecasts.

B. capabilities are not so tightly aligned as to offer little to no room for response flexibility. When assessing the potential for failure, operations should look to its line of fit concerning alignment to market needs. Too tight a fit risks market needs expanding beyond operational capabilities.

When outsourcing products or processes, a critical first step for all involved is identification of the company's: A. patents. B. core competencies. C. strategic plan. D. existing capacity strategy.

B. core competencies. Core competencies are the skills or knowledge sets that enable a firm to provide the greatest level of value to its customers in a way that is difficult for competitors to emulate and that provides for future growth.

Early due diligence concerning a backward integration initiative includes an analysis of both companies': A. cash flows. B. corporate cultures. C. balance sheets. D. supplier relationships.

B. corporate cultures. A corporate culture includes a set of assumptions and values that members of the company share. These common assumptions influence the way the company operates.

Forward integration occurs when a company: A. expands on its supply side. B. expands on its demand side. C. expands into overseas (foreign) markets. D. insources a previously outsourced operation.

B. expands on its demand side. Forward integration occurs when a company expands toward the end user or customer side.

In the five forces model of competition, as products available from rival suppliers become less differentiated, supplier rivalry is said to: A. become stagnant. B. increase. C. diminish design innovation. D. cause price increases.

B. increase. With similar product offerings, buyers have less reason to be brand-loyal, thus increasing supplier rivalry.

The process used to validate that adequate resources and capacity exist for the S&OP production plan is A. bill of resources. B. resource planning. C. rough-cut capacity planning. D. capacity requirements planning.

B. resource planning. Resource planning is the process that evaluates the availability of key resources and the feasibility of the production plan generated by the sales and operations plan. A bill of resources is incorrect, because it determines the amount of key resources used in the manufacturing process to produce a product and is a necessary input to the resource planning process. Rough-cut capacity planning is incorrect, because it is the process used by master scheduling to validate that capacity is adequate. Capacity requirements planning is incorrect because it is the process that validates the adequacy of capacity to execute the material plan.

Various cost-saving opportunities can be achieved from acquiring businesses with closely related: A. financial calendars and accounting practices. B. sales activities where products are sold to the same customers. C. workforce employees. D. philosophies concerning borrowing for capital expenditures.

B. sales activities where products are sold to the same customers. Various cost-saving opportunities spring from acquiring businesses with closely related sales and marketing activities where products are sold to the same customers. A single sales force can often reduce sales costs

The integration effort of cable companies to diversify into providing internet and telephone services is an example of: A. a company seeking to become a monopoly. B. strategic fit in R&D and technological activities. C. a larger company restraining industry opportunities. D. the need for more government regulations.

B. strategic fit in R&D and technological activities. Businesses with a strategic fit in R&D or technology activities perform better together than apart because of potential cost savings.

Product life cycle analysis concentrates on: A. the different versions of a product offering. B. the sales history of a product in stages. C. the product development cycle. D. the engineering changes made to a product's bill of material.

B. the sales history of a product in stages. A product life cycle is the stages a product goes through from beginning to end.

Reporting metrics on parts reduction, recycled content, and total weight reduction including packaging is a good example of how: A. the voice of the customer can facilitate savings. B. using life cycle analysis can result in resource and energy savings. C. design for logistics can benefit an organization holistically. D. component modularity can drive product design.

B. using life cycle analysis can result in resource and energy savings. Targets for reducing the number of parts, increasing recycled content, and reducing the overall weight of products are good examples of how using life cycle analysis has helped companies to identify where resource and energy reductions can be made.

What is the primary purpose of a core values statement? A. Satisfies the needs of legal and regulatory stakeholders B. Emphasizes to shareholders the organization's commitment to ethical behavior C. Provides ethical and business guidepost to organization members D. Enhances recruitment of the right kind of employees

C. Provides ethical and business guidepost to organization members

A B2B organization sells hydraulic presses that are priced at $25,000. The unit contribution margin (CM) is $10,000. If the organization has $4,000,000 in fixed costs, what is the break-even point in units? A. 160 units B. 267 units C. 400 units D. 1,600 units

C. 400 units The break-even point in units is calculated as the fixed costs divided by the unit CM. $4,000,000/$10,000 = 400 units.

A manufacturer of building supplies decides to improve its competitive advantage by buying sand and gravel suppliers near its factories. What type of strategy is this? A. Horizontal integration B. Forward integration C. Backward integration D. Disintermediation

C. Backward integration According to the APICSDictionary, backward integration is "the process of buying or owning elements of the production cycle and channel of distribution back toward raw material suppliers." When an organization is able to produce raw materials, parts, or services with at least the same efficiency, reliability, and quality as its suppliers, then backward integration may make sense.

A fairly new organization has no subsidiaries. It is considering creating two product lines, one that is the cheapest option on the market and another that is a premium line with high cost but innovative differentiating features. What is an issue with this strategy? A. The manufacturing functional area will need to generate synergy from the twin focus on cost control and quality. B. The organization will need to become a best-cost provider. C. Both employees and customers will be confused by the conflicting brand identities. D. The organization will need to develop a "dual operating system" for its functional areas to function effectively.

C. Both employees and customers will be confused by the conflicting brand identities. It is very challenging for one business unit to implement multiple strategies simultaneously. The brand identity will be difficult to overcome. Multiple marketing messages will confuse customers; different production and marketing goals will confuse employees.

Which of the following describes the environment that exists when international competition has become significant? A. Offshoring becomes a necessary alternative. B. Mass customization is frequently used as a product differentiator. C. Even small advantages in product and service specifications can have a significant impact on competitiveness. D. Operations must continuously apply cost-saving measures to all existing products to help increase market share.

C. Even small advantages in product and service specifications can have a significant impact on competitiveness. From a market perspective, international competition has become increasingly intense. Even small advantages in product and service specifications can have a significant impact on competitiveness.

Which generic strategy has two subsets? A. Differentiation B. Best-cost provider C. Focus D. Low-cost provider

C. Focus The focus strategy has two subsets: a low-cost focus and a differentiation focus.

A company has decided to redesign a product's overall value chain to eliminate some cost-producing activities. This product is currently in the company's broad differentiation strategy product group. What is the product likely moving to? A. Innovation product group B. Focused differentiation strategy product group C. Focused low-cost strategy product group D. Product group that will compete on delivery speed

C. Focused low-cost strategy product group A company competes on either an overall low-cost provider strategy, a broad differentiation strategy, a focused low-cost strategy, or a focused differentiation strategy.

Which of the following is the most appropriate response when a significant increase in production volume is anticipated? A. Obtaining a flexible manufacturing system B. Increasing the degree of vertical integration C. Realigning the process to match the new volume D. Implementing a focused factory concept

C. Realigning the process to match the new volume If demand requires an increase in production volume for a product, the first step is to investigate whether or not the process can support the anticipated volume.

As an organization becomes more horizontal, which of the following will occur? A. Decreasing exposure to integration and culture risk B. Neutral risk, because many suppliers provide the same components C. Increased exposure to supplier risk D. Need to expand the supplier base with redundant suppliers to reduce overall risks to supply chain performance

C. Increased exposure to supplier risk Increased reliance on suppliers in the supply chain increases exposure to supplier risk. Also, if the horizontal integration involves entering new global markets, this may increase risks related to cultural differences. If the horizontal integration requires new business partnerships, mergers, etc., this will increase integration risk. However, horizontal integration can increase diversification, which reduces overall business risk.

Which of the following is likely true of a new product development strategy? A. It will be used to augment manufacturing if there is significant uncertainty in operational delivery. B. It will be downsized if operations continuously beats standard times. C. It will be more robust and aggressive if accompanied by heightened confidence that operations is capable of delivering a quality product. D. It will take a back seat to product volumes if operations are highly efficient.

C. It will be more robust and aggressive if accompanied by heightened confidence that operations is capable of delivering a quality product. Operational knowledge is vital to establishing an operations-based strategic advantage. A superior understanding of what one's processes can do at the limits of their capability allows the organization to better provide products and services, due to confidence in knowing for sure that the processes can deliver. Well-controlled processes will also have fewer errors and waste and therefore higher efficiency (lower costs, higher quality).

"The voice of the customer" refers to identifying customer wants through which two of the following mechanisms? A. Working to resolve customer complaints and listening to requests from potential customers B. Listening to requests from potential customers and producing reliable products consistently C. Listening to requests from potential customers and conducting in-depth qualitative customer interviews D. Producing reliable products consistently and conducting in-depth qualitative customer interviews

C. Listening to requests from potential customers and conducting in-depth qualitative customer interviews "Listening to the voice of the customer" refers to the information marketing and product design need when creating new products. The idea is to design new products that reflect what the customer actually wants versus what marketing thinks they should have.

For a make-to-stock company, what strategy would be appropriate for growth in an existing market when the organization doesn't want to increase its aggregate inventory levels and its products are still in the growth phase? A. Product development B. Diversification C. Market penetration D. Market development

C. Market penetration In the Ansoff product-market growth matrix, four quadrants are formed using existing markets and new markets on one axis and existing products and new products on the other axis. The company wants to grow in existing markets, and this could involve either market penetration or product development. Product development would expand aggregate inventory if new product lines are added. Replacement products are also not needed due to the current products being in the growth phase. Therefore, market penetration is the growth strategy.

A B2B organization makes replaceable belts for newspaper printing press maintenance. When identifying their customer segments, who would be the economic buyer? A. Newspaper readers B. The Associated Press C. Newspaper companies D. Printing press manufacturers

C. Newspaper companies The economic buyer is the organization whose budget funds the purchase. Since these are maintenance items for products out in the field, the newspaper companies would be the primary buyers. The printing press manufacturers would also be buyers but would buy in lower volume.

Which of the following is critical information that is required during the operations strategic planning process? A. Prior year's net income statement B. Supply chain design C. Overall objectives of the organization D. Human resources plan for future hiring

C. Overall objectives of the organization One area's strategic plan must be aligned with the strategic plan of the overall company.

A company produces and distributes three families of basic materials that are sold directly to manufacturers. The industry is very competitive, and the growth rate is low. Which of the following would be the most appropriate focus for the operations strategy in this situation? A. Increasing operating flexibility B. Increasing speed of production C. Reducing waste in operations D. Reducing lead time variability

C. Reducing waste in operations The performance objectives for a company differ depending on the type of business. In this situation, with a business that produces standard, basic products in a competitive environment, the focus of the operations strategy will likely be cost reduction and efficiencies, which can be achieved by reducing waste in operations.

Which of the following represents a trend in supplier relationships? A. Increase in the number of transportation providers B. Decrease in the outsourced production of goods and services C. Reduction in the number of suppliers D. Fewer partnerships with customers

C. Reduction in the number of suppliers Shrinking the number of suppliers enables companies to form much closer supply channel relationships.

Which of the following includes an analysis of the nature of the firm, the market, and the business environment? . A. Benchmarking B. S&OP C. Strategic plan D. Product family analysis

C. Strategic plan S&OP is incorrect, because although it is concerned with these factors, the analysis is conducted as a part of the business planning process. Benchmarking is incorrect because it is a separate part of business planning, which does not include analysis of the nature of the firm, the market, or the business environment. Product family analysis is incorrect because it occurs as a part of S&OP and does not include analysis of the nature of the firm, the market, or the business environment.

A company with a product differentiation strategy is focused on: A. high productivity. B. mass customization. C. customer-valued product features. D. low cost of quality.

C. customer-valued product features. Product differentiation is a strategy of making a product distinct from the competition on a non-price basis such as availability, durability, quality, or reliability.

Management agrees that a good way to win sales for the organization's standard product, which competes in a highly variable demand market, is to switch from make-to-order to make-to-stock. The primary order winner for this product is most likely: A. product quality. B. design flexibility. C. delivery lead time. D. cost.

C. delivery lead time. In a make-to-stock environment, the delivery lead time would be from the time the order is received to the time the product reaches the customer. No manufacturing or assembly would be required, as there would be with a make-to-stock product.

In the five forces model of competition, if switching suppliers is relatively simple, then supplier rivalry is said to: A. inhibit innovation. B. enable risk pooling. C. increase. D. decrease.

C. increase. The less costly it is for buyers to switch purchases from one seller to another, the easier it is for sellers to steal customers from rivals.

A market leader in systems computing fails to adapt to market trends for smaller devices and cloud computing. This reveals a poor assessment process concerning its: A. monthly sales and operations planning (S&OP). B. strengths, weaknesses, opportunities, and threats (SWOT) analyses. C. operations strategy. D. executive dashboard capabilities.

C. operations strategy. The operations strategy shapes the long-term capabilities of an operation and should be consistent with the overall strategy.

Which of the following features is characteristic of continuous process manufacturing? A. Low-cost production B. Price as an order-winning criterion C. Narrow product range D. All of the above

D. All of the above Continuous manufacturing is defined as a type of manufacturing process that is dedicated to the production of a very narrow range of standard products. The rate of product change and new product information is very low. Significant investment in highly specialized equipment allows for a high volume of production at the lowest manufacturing cost. Thus, unit sales volumes are very large, and price is almost always a key order-winning criterion.

Which of the following types of processes best supports the widest range of production volume and permits a company to most easily adjust its capacity to match demand? A. Continuous B. Jobbing C. Line D. Batch

D. Batch Batch production is a manufacturing technique in which parts are accumulated and processed together in a lot. Batch processes also enable companies to easily adjust resources to produce different products.

Why should caution be exercised in using value chain analysis alone in efforts to meet or beat top competitors' costs? A. Value chain analysis often ignores purchasing. B. Competitors may spend more on sales and marketing; this is typically not revealed in value chain analysis. C. Financial and portfolio analysis are superior tools for cost reductions. D. Cost and price differences may emanate from supplier and distribution superiority.

D. Cost and price differences may emanate from supplier and distribution superiority. Cost and price differences between competitors may have their origins in activities performed by suppliers or by distribution allies.

A strategic decision has been made to expand a highly customer-centric organization into a foreign country. What should a solid risk assessment reveal? A. Locals should carry the major share of customer interactions. B. Local regulations will probably not be an issue, as the opportunity for new tax receipts usually wins the day. C. Processes that work in one country should work in another. D. Cultural interactions are likely a major source of potential failure.

D. Cultural interactions are likely a major source of potential failure. When a U.S.-based amusement company decided to expand its largely successful theme park operations to Europe, it brought American management styles, cultural tastes, labor practices, and marketing pizzazz to Europe. When the French stayed away in droves, the company accused them of cultural snobbery.

Which is an order winner characteristic that is typical in the growth phase of a product or service's life cycle and that is challenging to maintain in this life cycle phase when sales are growing exponentially? A. Competitive price B. Flexibility C. Quality D. Dependability

D. Dependability During the growth phase, a key order winner tends to be dependability, because a product needs to be available online/at a desired location and/or available within the desired lead time. This can be a challenge if the growth rate is steep.

Which of the following performance objectives refers to the ability to keep promises to customers? A. Speed B. Cost C. Speed D. Dependability

D. Dependability There are five generic performance objectives: quality, speed, dependability, flexibility, and cost. Dependability refers to keeping promises to customers. These promises can include factors such as time and volume, and the ability to delivery the correct amount in the required time frame.

What is the most common form of international distribution? A. Expansion into foreign markets through warehousing B. Establishing a joint venture with a foreign organization C. Licensing international companies for manufacturing processes D. Export of products into foreign markets

D. Export of products into foreign markets All of these are ways to expand into foreign markets, but exporting is by far the most common and the least expensive.

Which if the following performance objectives refers to an operation's ability to offer different configurations of a product and react to requests for changes from a customer? A. Quality B. Speed C. Cost D. Flexibility

D. Flexibility There are five generic performance objectives: quality, speed, dependability, flexibility, and cost. Flexibility has two dimensions: volume/mix and agility. That is, an operation's ability to produce different types and/or varieties of products or services, and also how quickly the operation is able to respond to requested changes.

What is a normal progression for a unique product characteristic that delights customers? A. it moves from an order qualifier to eventually become an order winner. B. It always starts off at the product introduction stage as the focal point for advertising and sales presentations and always moves to the growth stage. C. It becomes a likely catalyst for a line extension or a new product. D. It starts as an order winner and eventually becomes an order qualifier.

D. It starts as an order winner and eventually becomes an order qualifier. Such characteristics rely on their novelty, and they're no longer novel once they are offered in the market. What was an order winner may in time become an order qualifier if it starts to be considered a standard requirement.

What is the term for an alliance between two companies to share the risk of equity capital in a business opportunity? A. Line of credit B. Limited partnership C. Marquis partnership D. Joint venture

D. Joint venture A joint venture is an agreement between two or more firms to risk equity capital to attempt a specific business objective.

An organization's demand management professionals indicate that there will be a sales promotion at the end of the month for one of the organization's products. The product is in maturity. A substitute product was just introduced by a subsidiary, and marketing mentions something about potential for product cannibalization. What should operations professionals ask more about? A. Likelihood of convincing the subsidiary to retire their harmful product B. Timing of when to retire the mature product C. Amount of dollar spend on the sales promotion D. Likely impact on demand of the product cannibalization

D. Likely impact on demand of the product cannibalization Questions could include the likely impact of the product cannibalization and of the sales promotion. Asking about the dollar spend would be less useful because it is demand management's responsibility to estimate this impact.

A company with a long history in the marketplace is considering acquiring a start-up that has technology the company believes will expand its business. What is one of the major obstacles the company will have to overcome if the acquisition occurs? A. Explaining the acquisition to the financial media B. Bringing the new employees into the 401(k) plan C. Explaining the acquisition to the board of directors D. Mismatch of organizational cultures

D. Mismatch of organizational cultures There may be a mismatch of organizational cultures, which can extend from the leaders downward through the entire organization. A workforce used to more decision-making authority and fewer rules will not adapt smoothly to a more regimented organization. The other answers are insignificant compared to the culture issue.

What is an accurate description of order winning and order qualifying factors? A. Performance must be above a certain level in at least three performance objective categories for an order winning threshold to be attained. B. Quality must always be one of the order winning factors of performance. C. There can be only one order winning factor involved in a customer's decision to purchase a product or service. D. Order qualifying factors represent minimal requirements, while order winning factors exceed expectations.

D. Order qualifying factors represent minimal requirements, while order winning factors exceed expectations. Order qualifying factors represent minimal requirements, while order winning factors exceed expectations. The other answer choices define the terms too precisely. For example, quality may or may not be the order winning factor, and the number of factors may vary.

Which of the following is true of internal initiatives that enhance existing order qualifiers? A. They should be driven by the voice of the customer. B. They will likely increase business. C. They are most appropriately prioritized using customer feedback. D. They are unlikely to provide much competitive benefit.

D. They are unlikely to provide much competitive benefit. Any further improvement in qualifying factors above the qualifying level is unlikely to gain much competitive benefit.

Customer-supplier relationships that focus on the future and long-term intercompany team building may be: A. something that needs to be formalized using long-term partnership agreements. B. jeopardized by a failure to meet expected order qualifying characteristics. C. jeopardized by a failure to meet expected order winning characteristics. D. a major order winning factor.

D. a major order winning factor. Enlightened customer-supplier relationships that focus on the big picture and the longer term may be a major order winning factor.

The voice of the customer process is in complete alignment with: A. process flow analysis methodology. B. the Delphi method. C. qualitative forecast techniques. D. quality function deployment (QFD).

D. quality function deployment (QFD). The voice of the customer involves actual customer descriptions of the functions and features customers desire for goods and services. It relates to QFD.

A company successfully incentivizes employees in a labor-paced environment to exceed scheduled output in scheduled time. This is an example of an employee wage policy: A. That will be totally compatible with almost every unionized organization. B. that is bound to cause strife when engineering increases standard times by operation. C. that will likely put the company's products at the top in this industry for quality. D. that is creating production value.

D. that is creating production value. When designing the payroll system at one steel company, management decided to incentivize the employees with a 1% bonus rewarded for every 1% that their output exceeded the goals. The company consistently achieved their low-cost provider goals.

Organizations consisting of a collection of companies that have similar and complementary supply chains will likely perform better together because of: A. their ability to corner the market for certain component parts and raw materials, driving prices down. B. their leverage with municipalities that depend on them for jobs and taxes. C. the capability to move inventories off the books in transit between various business entities. D. the potential for skills transfer and/or volume discounts in procuring materials, resource sharing, and logistics.

D. the potential for skills transfer and/or volume discounts in procuring materials, resource sharing, and logistics. Diversified corporations with multiple businesses that have a strategic fit with respect to their supply chain activities can perform better together because of the potential for skills transfer and/or volume discounts in procuring materials, resource sharing, and logistics.


Related study sets

Karch Pharmacology CH55 - Lower Respiratory Drugs

View Set

Life Only, Chapter 5 - A. Insurance Commissioner

View Set

Old Testament Exam 1 Springer Study Guide

View Set

Sudden Cardiac Death in Athletes

View Set